Understanding the Derivative of Inverse Trig Functions

Click For Summary
The discussion focuses on understanding the derivative of the arcsine function when substituting variables in integration. It highlights that when differentiating the arcsine of (u/a), an additional factor appears in the numerator. To resolve this, a variable substitution is recommended, replacing u/a with x, which transforms the integrand into the desired form of 1/√(1-x²). This substitution process involves additional changes that effectively cancel out the factor of 1/a. Ultimately, this approach leads to a clearer and more manageable result in the integration process.
alijan kk
Messages
130
Reaction score
5

Homework Statement


question.PNG

why this formula works ?

Homework Equations

The Attempt at a Solution


when i take the derivative of the right side ,,, there is an additional "a" in the numerator in place of 1,, why the derivative of arcsine of (u/a) not exactly same with the expression under the integral sign
28217670_236535466916604_1075396015_o.jpg
 

Attachments

  • question.PNG
    question.PNG
    5.6 KB · Views: 679
  • 28217670_236535466916604_1075396015_o.jpg
    28217670_236535466916604_1075396015_o.jpg
    24.7 KB · Views: 323
Last edited:
Physics news on Phys.org
In order to finally perform the integration in the last step you need to do a substitution, say replacing ##u/a## by ##x## so that your integrand has the form ##1/\sqrt{1-x^2}##. In order to do that you need to make the other changes that are part of doing a variable substitution in integration. Those changes should cancel out the factor ##1/a## and leave you with the nice, clean result you want.
 
  • Like
Likes alijan kk
now i understand it very well,,,, thankyou for they reply
andrewkirk said:
In order to finally perform the integration in the last step you need to do a substitution, say replacing ##u/a## by ##x## so that your integrand has the form ##1/\sqrt{1-x^2}##. In order to do that you need to make the other changes that are part of doing a variable substitution in integration. Those changes should cancel out the factor ##1/a## and leave you with the nice, clean result
 
Question: A clock's minute hand has length 4 and its hour hand has length 3. What is the distance between the tips at the moment when it is increasing most rapidly?(Putnam Exam Question) Answer: Making assumption that both the hands moves at constant angular velocities, the answer is ## \sqrt{7} .## But don't you think this assumption is somewhat doubtful and wrong?

Similar threads

  • · Replies 2 ·
Replies
2
Views
1K
Replies
16
Views
2K
  • · Replies 7 ·
Replies
7
Views
1K
  • · Replies 12 ·
Replies
12
Views
3K
Replies
4
Views
2K
  • · Replies 14 ·
Replies
14
Views
2K
  • · Replies 3 ·
Replies
3
Views
1K
Replies
1
Views
1K
  • · Replies 3 ·
Replies
3
Views
3K
  • · Replies 2 ·
Replies
2
Views
2K